Dirac Lagr in Hermitian form

In summary, the Dirac lagrangian can be written in a symmetric form by adding the hermitian conjugate, which leads to the same equations of motion. There should also be a connection between the two lagrangians via a 4divergence, which can be achieved by choosing a specific form for the additional term K^{\mu}. However, this results in the initial lagrangian being doubled and the need for a - sign in the final expression.
  • #1
ChrisVer
Gold Member
3,378
464
Is there any way to write the Dirac lagrangian to have symmetric derivatives (acting on both sides)? Of course someone can do that by trying to make the Lagrangian completely hermitian by adding the hermitian conjugate, and he'll get the same equations of motion (a 1/2 must exist in that case)...However same equations of motion, imply that there should be a connection between the two Lagrangians via a 4divergence, right? Unfortunately I cannot see what that could be.
[itex]L_{D}= \bar{\psi} (i γ^{\mu}∂_{\mu}-m) \psi [/itex]
In particular choosing
[itex]L_{D}= \bar{\psi} (i γ^{\mu}∂_{\mu}-m) \psi + ∂_{\mu}K^{\mu}[/itex]
seems to bring some result if I choose [itex]K^{\mu}=i\bar{\psi}\gamma^{\mu}\psi[/itex]
but the initial lagrangian seems to get doubled...
 
Physics news on Phys.org
  • #2
[itex]L_{D}=\frac{1}{2} \bar{\psi} i γ^{\mu}∂_{\mu} \psi + \frac{1}{2} \bar{\psi} i γ^{\mu}∂_{\mu} \psi -m \bar{\psi}\psi[/itex]

[itex]L_{D}=\frac{1}{2} \bar{\psi} i γ^{\mu}∂_{\mu} \psi + \frac{1}{2} i ∂_{\mu}(\bar{\psi}γ^{\mu}\psi) - \frac{1}{2}i (∂_{\mu}\bar{\psi})\gamma^{\mu}\psi -m \bar{\psi}\psi[/itex]

[itex]L_{D}=\frac{1}{2} \bar{\psi} (i γ^{\mu}[∂_{\mu}^{→}-∂_{\mu}^{←}] -2m) \psi + \frac{1}{2} i ∂_{\mu}(\bar{\psi}γ^{\mu}\psi)[/itex]

Should the - exist?
 
Last edited:
  • #3
Yes, your last expression is right.
 

1. What is the Dirac Lagrangian in Hermitian form?

The Dirac Lagrangian in Hermitian form is a mathematical expression that describes the dynamics of a quantum system with spin. It is a function of the position and momentum of the particles in the system, and it takes into account the effects of spin on the particles' motion. It is a fundamental component of the Dirac equation, which describes the behavior of fermions, such as electrons, in relativistic quantum mechanics.

2. What is the significance of the Hermitian form in the Dirac Lagrangian?

The Hermitian form of the Dirac Lagrangian is important because it ensures that the resulting equations of motion are self-consistent and that the probabilities of all possible outcomes add up to one. This is necessary for the mathematical consistency of quantum mechanics and allows us to make accurate predictions about the behavior of quantum systems.

3. How is the Dirac Lagrangian in Hermitian form derived?

The Dirac Lagrangian in Hermitian form is derived by applying the principle of least action to the relativistic wave equation for fermions, known as the Dirac equation. This involves minimizing the action, which is a mathematical expression that describes the energy and momentum of a system, with respect to the position and momentum variables in the equation. The resulting Lagrangian is then expressed in the Hermitian form to ensure the self-consistency of the equations.

4. What are the applications of the Dirac Lagrangian in Hermitian form?

The Dirac Lagrangian in Hermitian form is used in many areas of physics, including particle physics, quantum field theory, and condensed matter physics. It is the basis for the Standard Model of particle physics, which describes the interactions between fundamental particles and is essential for understanding the behavior of matter at the smallest scales. It also has applications in quantum computing and quantum information theory.

5. Are there any limitations to the Dirac Lagrangian in Hermitian form?

While the Dirac Lagrangian in Hermitian form is a very powerful tool for describing the dynamics of quantum systems, it has some limitations. For example, it does not take into account the effects of gravity and is not applicable to systems with zero or integer spin. Additionally, it only describes non-interacting particles and cannot account for the interactions between particles. These limitations have led to the development of more complex Lagrangians, such as the Yang-Mills Lagrangian, which can describe interacting particles and include the effects of gravity.

Similar threads

  • High Energy, Nuclear, Particle Physics
Replies
3
Views
878
  • High Energy, Nuclear, Particle Physics
2
Replies
49
Views
4K
  • High Energy, Nuclear, Particle Physics
Replies
1
Views
1K
  • High Energy, Nuclear, Particle Physics
2
Replies
38
Views
3K
  • High Energy, Nuclear, Particle Physics
Replies
4
Views
2K
  • High Energy, Nuclear, Particle Physics
Replies
4
Views
1K
  • Advanced Physics Homework Help
Replies
0
Views
658
  • Advanced Physics Homework Help
Replies
1
Views
872
  • High Energy, Nuclear, Particle Physics
Replies
13
Views
3K
Back
Top